876 views
1 votes
1 votes
Eric throws two dice, and his score is the sum of the values shown.  Sandra throws one die, and her score is the square of the value shown.  What is the probability that Sandra’s score will be strictly higher than Eric’s score?

a.  137/216

b.  17/36

c.  173/216

d.  5/6

2 Answers

2 votes
2 votes

All possible combination= 6*6*6 = 216

If Sandra's dice=1 => her score=1 ==>no winning case

If Sandra's dice=2 => her score=4 ==> wins if Eric dice= (1,1) (1,2) (2,1) ==>3 ways

If Sandra's dice=3 => her score=6 ==> wins if Eric dice= (1,1)..(1,6);(2,1)..(2,6);(3,1)..(3,5);(4,1)..(4,4);(5,1)..(5,3);(6,1)..(6,2)

==>(6+6+5+4+3+2)=26 ways

If Sandra's dice=4 or 5 or 6 => her score=16 or 25 or 36 >12(max Eric's score) ==> 36+36+36=108 ways

thus, total winning cases for Sandra= (3+26+108) =137 ways

thus,  probability that Sandra’s score will be strictly higher than Eric’s score = 137/216

0 votes
0 votes

Sorry for that i read question wrongly ( sandra also throws 2 dices )... But for my satisfaction i keep it as it is

DICE = {1,2,3,4,5,6}

Sum possible with rolling Two dices = {2,3,4,5,6,7,8,9,10,11,12}

favorable cases for

SUM = 2 ------ { (1,1) } =====> $\frac{1}{36}$

SUM = 3 ------ { (1,2),(2,1) } =====> $\frac{2}{36}$

SUM = 4 ------ { (1,3),(2,2),(3,1) } =====> $\frac{3}{36}$

SUM = 5 ------ { (1,4),(2,3),(3,2),(4,1) } =====> $\frac{4}{36}$

SUM = 6 ------ { (1,5),(2,4),(3,3),(4,2),(5,1) } =====> $\frac{5}{36}$

SUM = 7 ------ { (1,6),(2,5),(3,4),(4,3),(5,2),(6,1) } =====> $\frac{6}{36}$

SUM = 8 ------ { (2,6),(3,5),(4,4),(5,3),(6,2) } =====> $\frac{5}{36}$

SUM = 9 ------ { (3,6),(4,5),(5,4),(6,3) } =====> $\frac{4}{36}$

SUM = 10 ------ { (4,6),(5,5),(6,4) } =====> $\frac{3}{36}$

SUM = 11 ------ { (5,6),(6,5) } =====> $\frac{2}{36}$

SUM = 12 ------ { (6,6) } =====> $\frac{1}{36}$

 

Eric Sum would be {2,3,4,5,6,7,8,9,10,11,12}

when Eric Sum=2, Sandra's Sum should be grater than 2

when Eric Sum=3, Sandra's Sum should be grater than 3 and etc..

 

Required Probability

= { ( Eric=2 and Sandra >2 ) + ( Eric=3 and Sandra >3 ) + ( Eric=4 and Sandra >4 ) + ...... + ( Eric=12 and Sandra >12 ) }

= $ ( \frac{1}{36}  * \frac{35}{36} )+ ( \frac{2}{36}  * \frac{33}{36} )+ ( \frac{3}{36}  * \frac{30}{36} )+ ( \frac{4}{36}  * \frac{26}{36} )+ ( \frac{5}{36}  * \frac{21}{36} )+ ( \frac{6}{36}  * \frac{15}{36} )+ ( \frac{5}{36}  * \frac{10}{36} )+ ( \frac{4}{36}  * \frac{6}{36} )+ ( \frac{3}{36}  * \frac{3}{36} )+ ( \frac{2}{36}  * \frac{1}{36} )+ ( \frac{1}{36}  * \frac{0}{36} ) $

= $ \frac{1}{36^2} $ . {(35*1)+(33*2)+(30*3)+(26*4)+(21*5)+(15*6)+(10*5)+(6*4)+(3*3)+(2*1)+(1*0)}

= $ \frac{1}{1296} $ . {(35)+(66)+(90)+(104)+(105)+(90)+(50)+(24)+(9)+(2)+(0)}

= $ \frac{1}{1296} $ . {(575)}

= $ \frac{575}{1296} $ 

 

edited by

Related questions

0 votes
0 votes
1 answer
1
Raj Kumar 7 asked Jul 19, 2018
1,482 views
The number of integer n between 100 < n <300 such that 16 divides (n^2-n-2) and 25 divides (n^2+2*n-3).
0 votes
0 votes
1 answer
2
Raj Kumar 7 asked Jul 13, 2018
857 views
Q. Two consecutive numbers are removed from the progression 1, 2, 3, ...n. The arithmetic mean of the remaining numbers is 26 1/4. The value of n isa. 60 ...
0 votes
0 votes
1 answer
3
farhan777 asked 3 days ago
30 views
how to check the validity of an a argument using laws of logics
0 votes
0 votes
1 answer
4
Anand67222 asked Oct 14, 2023
322 views
How many simple directed (unweighted) graphs on the set of vertices {v0,v1,…v5} are there that have at most one edge between any pair of vertices? (That is, for two ver...